Classify the origin of the system and draw the phase portrait

Click For Summary
The system described by the equations has eigenvalues of -1 and 3, indicating that the origin is classified as a saddle point due to the differing signs of the eigenvalues. The general solution of the system is derived from the eigenvectors associated with these eigenvalues, leading to a combination of exponential functions. The phase portrait can be drawn to illustrate the flow in all four quadrants, highlighting the saddle behavior at the origin. Additional clarification on the classification of the origin as a saddle point is warranted, as it reflects the nature of the eigenvalues. Overall, the analysis and proposed phase portrait effectively represent the system's dynamics.
bjohnson2001
Messages
13
Reaction score
0

Homework Statement



Draw the phase portrait and classify the origin of the system:

xdot = [1 2; 2 1]x

Homework Equations



characteristic equation:

det(A-lambda*I) = 0

The Attempt at a Solution



First find the eigenvalues and eigenvectors:

det(A-lambda*I) = (lambda+1)(lambda-3) = 0


we can see that the eigenvalues are:

lambda_1 = -1 and lambda_2 = 3

for lambda_1 = -1: 2*k1 + 2*k2 = 0
k1 = -k2

when k1 = 1, k2 = - 1 the related eigenvector is (1; -1)


for lambda_2 = 3: -2*k1 + 2*k2 = 0
k1 = k2

when k1 = 1, k2 = 1 the eigenvector is K2 = (1; 1)

since the matrix of coefficeints is a 2x2 matrix and since we found two linearly independent solutions,

the general solution of the system is:

X = c1*X1 + c2*X2 = c1*(1;-1)*exp(-t) + c2*(1; 1)*exp(t)
or
x = c1*exp(-t) + c2*exp(t)
y = -c1*exp(-t) + c2*exp(t)

We can classify the origin as neither a repeller nor an attractor.

Is this correct?

Also I feel like I need to provide more information when classifying the origin but I don't know what. For instance should I call the origin a saddle point because it has eigenvalues of different polarities?
 

Attachments

  • phase plane.png
    phase plane.png
    5.3 KB · Views: 568
Physics news on Phys.org
The origin is a saddle. Also, I think you should draw a better phase portrait showing the flow in all four quadrants. Here's the code if you wish to do so in Mathematica but I don't think you can run it in Alpha:

StreamPlot[{x+2y,2x+y},{x,-2,2},{y,-2,2}]
 
Question: A clock's minute hand has length 4 and its hour hand has length 3. What is the distance between the tips at the moment when it is increasing most rapidly?(Putnam Exam Question) Answer: Making assumption that both the hands moves at constant angular velocities, the answer is ## \sqrt{7} .## But don't you think this assumption is somewhat doubtful and wrong?

Similar threads

  • · Replies 1 ·
Replies
1
Views
2K
Replies
12
Views
2K
  • · Replies 4 ·
Replies
4
Views
1K
  • · Replies 2 ·
Replies
2
Views
1K
  • · Replies 10 ·
Replies
10
Views
2K
  • · Replies 11 ·
Replies
11
Views
3K
Replies
8
Views
3K
  • · Replies 2 ·
Replies
2
Views
2K
  • · Replies 5 ·
Replies
5
Views
2K
  • · Replies 3 ·
Replies
3
Views
3K